|x-6|>5

This topic has expert replies
Legendary Member
Posts: 510
Joined: Thu Aug 07, 2014 2:24 am
Thanked: 3 times
Followed by:5 members

|x-6|>5

by j_shreyans » Fri Sep 05, 2014 11:03 pm
Is |x−6|>5 ?

(1) x is an integer

(2) x^2<1

OAB

User avatar
GMAT Instructor
Posts: 15539
Joined: Tue May 25, 2010 12:04 pm
Location: New York, NY
Thanked: 13060 times
Followed by:1906 members
GMAT Score:790

by GMATGuruNY » Sat Sep 06, 2014 12:14 am
j_shreyans wrote:Is |x−6|>5 ?

(1) x is an integer

(2) x^2<1

OAB
|a-b| = the DISTANCE between a and b on the number line.
Question stem, rephrased:
Is the distance between x and 6 greater than 5?

For x to be more than 5 places from 6, it must lie in one of the two RED RANGES shown below:
-----x-----1-----------6-----------11-----x-----

Statement 1: x is an integer
No way to determine whether x lies within one of the two red ranges.
INSUFFICIENT.

Statement 2: x²<1
For x² to be less than 1, -1 < x < 1.
Thus, x must lie in the red range to the left of 1 on the number line.
SUFFICIENT.

The correct answer is B.
Private tutor exclusively for the GMAT and GRE, with over 20 years of experience.
Followed here and elsewhere by over 1900 test-takers.
I have worked with students based in the US, Australia, Taiwan, China, Tajikistan, Kuwait, Saudi Arabia -- a long list of countries.
My students have been admitted to HBS, CBS, Tuck, Yale, Stern, Fuqua -- a long list of top programs.

As a tutor, I don't simply teach you how I would approach problems.
I unlock the best way for YOU to solve problems.

For more information, please email me (Mitch Hunt) at [email protected].
Student Review #1
Student Review #2
Student Review #3

Legendary Member
Posts: 510
Joined: Thu Aug 07, 2014 2:24 am
Thanked: 3 times
Followed by:5 members

by j_shreyans » Sat Sep 06, 2014 2:13 am
Hey ,

I was solving like below:

|x-6|>5

x-6>5
x>11 -------

and |x-6|>5
-x+6>5
-x>-1
x<1 -----right?

Statement 1 is not sufficient

Statement 2 -- x^2<1

So x<1 and x<-1

then what should be my next step?????


Pls let me know.

GMAT/MBA Expert

User avatar
GMAT Instructor
Posts: 16207
Joined: Mon Dec 08, 2008 6:26 pm
Location: Vancouver, BC
Thanked: 5254 times
Followed by:1268 members
GMAT Score:770

by Brent@GMATPrepNow » Sat Sep 06, 2014 5:54 am
j_shreyans wrote:Is |x−6| > 5 ?

(1) x is an integer

(2) x² < 1

OAB
When solving inequalities involving ABSOLUTE VALUE, there are 2 things you need to know:
Rule #1: If |something| < k, then -k < something < k
Rule #2: If |something| > k, then EITHER something > k OR something < -k
Note: these rules assume that k is positive


Target question: Is |x−6| > 5?

This is a great candidate for rephrasing the target question.
Aside: We have a free video with tips on rephrasing the target question: https://www.gmatprepnow.com/module/gmat- ... cy?id=1100

We'll apply rule #2 (above). If |x−6| > 5, then EITHER x−6 > 5 OR x-6 < -5
Simplify to conclude that EITHER x > 11 OR x < 1

So, we can REPHRASE the target question as....
REPHRASED target question: Is EITHER x > 11 OR x < 1? (this is a YES/NO question)

Statement 1: x is an integer
There are several values of x that satisfy this condition. Here are two:
Case a: x = 7, in which case the answer to the REPHRASED target question is NO, x is not either greater than 11 OR less than 1
Case b: x = 20, in which case the answer to the REPHRASED target question is YES, x is either greater than 11 OR less than 1
Since we cannot answer the REPHRASED target question with certainty, statement 1 is NOT SUFFICIENT

Statement 2: x² < 1
This tells us that -1 < x < 1
This means that x is definitely less than 1.
So, the answer to the REPHRASED target question is YES, x is either greater than 11 or less than 1
Since we can answer the REPHRASED target question with certainty, statement 2 is SUFFICIENT

Answer = B

Cheers,
Brent

For even more information on rephrasing the target question, you can read this article I wrote for BTG: https://www.beatthegmat.com/mba/2014/06/ ... t-question
Brent Hanneson - Creator of GMATPrepNow.com
Image

GMAT/MBA Expert

User avatar
GMAT Instructor
Posts: 16207
Joined: Mon Dec 08, 2008 6:26 pm
Location: Vancouver, BC
Thanked: 5254 times
Followed by:1268 members
GMAT Score:770

by Brent@GMATPrepNow » Sat Sep 06, 2014 5:56 am
j_shreyans wrote:Hey ,

I was solving like below:

|x-6|>5

x-6>5
x>11 -------

and |x-6|>5
-x+6>5
-x>-1
x<1 -----right?

Statement 1 is not sufficient

Statement 2 -- x^2<1

So x<1 and x<-1

then what should be my next step?????


Pls let me know.
This looks good, except it's unclear what role your calculations are playing.
You have taken the target question |x-6|>5 and concluded that EITHER x>11 OR x<1
This allows you to rewrite the target question as "Is it the case that x is either greater than 11 or less than 1?"

See my solution above for more detail.

Cheers,
Brent
Brent Hanneson - Creator of GMATPrepNow.com
Image

User avatar
Legendary Member
Posts: 1100
Joined: Sat May 10, 2014 11:34 pm
Location: New Delhi, India
Thanked: 205 times
Followed by:24 members

by GMATinsight » Sat Sep 06, 2014 9:36 am
j_shreyans wrote:Is |x−6|>5 ?

(1) x is an integer

(2) x^2<1

OAB
Question : Is |x−6|>5 ?
i.e. Is +(x−6)>5 ?
Is x−6>5 or -(x-6)>5?
Is x>11 or (x-6)<-5?
Question : Is x>11 or (x)<1?

Statement 1) x is an integer
NOT SUFFICIENT

Statement 2) x^2<1
i.e. -1 < x < 1
SUFFICIENT

Answer: Option B
"GMATinsight"Bhoopendra Singh & Sushma Jha
Most Comprehensive and Affordable Video Course 2000+ CONCEPT Videos and Video Solutions
Whatsapp/Mobile: +91-9999687183 l [email protected]
Contact for One-on-One FREE ONLINE DEMO Class Call/e-mail
Most Efficient and affordable One-On-One Private tutoring fee - US$40-50 per hour

User avatar
Legendary Member
Posts: 1100
Joined: Sat May 10, 2014 11:34 pm
Location: New Delhi, India
Thanked: 205 times
Followed by:24 members

by GMATinsight » Sat Sep 06, 2014 9:38 am
Hi J_Shreyan,

Please make sure that you post the questions in the correct thread.

P.S. YOU HAVE BEEN POSTING DATA SUFFICIENT QUESTIONS IN PROBLEM SOLVING THREAD
"GMATinsight"Bhoopendra Singh & Sushma Jha
Most Comprehensive and Affordable Video Course 2000+ CONCEPT Videos and Video Solutions
Whatsapp/Mobile: +91-9999687183 l [email protected]
Contact for One-on-One FREE ONLINE DEMO Class Call/e-mail
Most Efficient and affordable One-On-One Private tutoring fee - US$40-50 per hour

GMAT/MBA Expert

User avatar
Elite Legendary Member
Posts: 10392
Joined: Sun Jun 23, 2013 6:38 pm
Location: Palo Alto, CA
Thanked: 2867 times
Followed by:511 members
GMAT Score:800

by [email protected] » Sat Sep 06, 2014 11:34 am
Hi j_shreyans,

This question is perfect for TESTing Values.

We're asked if |X-6| > 5. This is a YES/NO question.

Fact 1: X is an integer.

If X = 6, then the answer to the question is NO
If X = 100, then the answer to the question is YES
Fact 1 is INSUFFICIENT

Fact 2: X^2 < 1

Here, we have a really limited range....

-1 < X < 1

No matter what we pick for X, the answer remains consistent...
If X = .99, then the answer to the question is YES
If X = 0, then the answer to the question is YES
If X = -.99, then the answer to the question is YES
Fact 2 is SUFFICIENT.

Final Answer: B

GMAT assassins aren't born, they're made,
Rich
Contact Rich at [email protected]
Image